Answer:
x = 32°
Step-by-step explanation:
∆KLM is an isosceles triangle because it has two equal sides, KL & KM. Therefore, the angles opposite to each of the two equal sides, which are referred to as the base angles are congruent to each other.
m<KML = m<KLM = 58°
m<MKL = 180 - (58 + 58) (Sum of triangle)
m<MKL = 64°
m<JKM = 180 - m<MKL (linear pair theorem)
m<JKM = 180 - 64 (Substitution)
m<JKM = 116°
∆JKM is also an isosceles triangle with two equal sides. Therefore, it's based angles (x & <J) would also be equal to each other.
Thus:
x = ½(180 - m<JKM)
x = ½(180 - 116) (Substitution)
x = 32°
Answer:
A number line going from 27 to 37, open circle at 32, everything to the right is shaded
Step-by-step explanation:
-1/2p < -16
multiply each side by -2 (don't forget to switch inequality symbol)
p > 32
We apply the Pythagorean theorem twice and obtain:
12 ^ 2 = x ^ 2 + (15-d) ^ 2
9 ^ 2 = x ^ 2 + d ^ 2
We observe that it is a system of two equations with two unknowns whose solutions are:
(x, d) = (-36/5, 27/5)
(x, d) = (36/5, 27/5)
We ignore the negative solution, therefore, the solution is:
(x, d) = (36/5, 27/5)
Answer:
The length of the new fence is:
x = 36/5 meters
45.3297 rounded to the nearest tenth is 45.3
3 is in the tenths place, so you look at the number to the right of it. If that number is 5 or more, you round up. If the number is 4 or less, you round down. So you round 45.3297 down to 45.3
<h2>Solution (a) :-</h2>
Let the cost of the less expensive comic book be x .
Then :-

Using this equation , we can find the cost of the less expensive comic book .
Price of the less expensive book :-





Cost of the less expensive book is 107 .
Therefore , the epice of the less expensive book = $ 107 .
<h2>Solution (b) :-</h2>
Cost of the more expensive book = 11x
Which means :-






Therefore , the price of the more expensive book = $ 1177 .